Difference between revisions of "Talk:2011 AMC 12B Problems/Problem 9"

(Created page with "Is the problem flawed? The range of numbers [-20, 10] includes 31 total numbers: [-1, -20], 0, and [1, 10]. Which makes the probabilities drastically different. For example the p...")
(No difference)

Revision as of 01:38, 21 December 2011

Is the problem flawed? The range of numbers [-20, 10] includes 31 total numbers: [-1, -20], 0, and [1, 10]. Which makes the probabilities drastically different. For example the probability for two consecutive negative numbers being chosen becomes $\frac{20}{31} \cdot \frac{20}{31} = \frac{400}{961}$